LSAT and Law School Admissions Forum

Get expert LSAT preparation and law school admissions advice from PowerScore Test Preparation.

User avatar
 Dave Killoran
PowerScore Staff
  • PowerScore Staff
  • Posts: 5853
  • Joined: Mar 25, 2011
|
#88292
Complete Question Explanation
(The complete setup for this game can be found here: lsat/viewtopic.php?f=177&p=88290#p88290)

The correct answer choice is (B).

This Global question asks you to identify the member that S can never be teamed with. While we did not identify S in a not-block inference during the setup, this is not a concern. Occasionally there are inferences that will not be obvious during the setup. In these cases, simply look to solve the problem using the usual methods. In this case, a Global question concerning a single variable is a perfect opportunity to use previous work. If we can find other problems that produced hypotheticals where S was teamed with one of the variables in the answer choices, then we can eliminate that answer from consideration. Let’s use that approach to knock out several answer choices:

In question #18, we produced a viable hypothetical in answer choice (E) that pairs S with P. Thus, P and S can be teammates and answer choice (D) can be eliminated. Similar hypotheticals pairing S and P were produced in questions #20 and #22.
In question #19, S is on the first-place team, and O can be on the first- or third-place team. Thus, O can team with S, and answer choice (C) can be eliminated. A similar hypothetical in question #21 shows the teaming of O and S.
Also in question #19, S is on the first-place team, and M can be on the first- or third-place team. Thus, M can team with S, and answer choice (A) can be eliminated. Similar hypotheticals in questions #21 and #22 also show the possible teaming of M and S.
Thus, by looking back at work we already completed, we can quickly and decisively eliminate three of the four answer choices, leaving only answer choices (B) and (E) in contention. The correct move at this point is to simply pick one and see what happens when you attempt to make a viable hypothetical using that variable.

In answer choice (B), when S and N are teammates, they must both be on the first-place team (S cannot be on the third-place team, and there is not enough room for both S and N on the second-place team). But, N on the first-place team violates the fourth rule which states that P N. Thus, P and N can never be on the same team, and answer choice (B) is correct.

In answer choice (E), T can be teamed with S. This is the hypothetical that proves this point:

G4-Q23-d1.png
You do not have the required permissions to view the files attached to this post.

Get the most out of your LSAT Prep Plus subscription.

Analyze and track your performance with our Testing and Analytics Package.